4
$\begingroup$

Let $\Omega$ be a bounded domain of $R^d$ with Lipschitz boundary. If $m>\frac{d}{2}$, such that $H^m(\Omega)$ is continuously embedded in $L^\infty(\Omega)$. Is $L^1(\Omega)$ continuously embedded in the dual space of $H^m(\Omega)$? Thank you very much.

$\endgroup$

1 Answer 1

4
$\begingroup$

Yes. This is a special case of the following result.

Proposition. Let $X$ and $Y$ be normed spaces and let $i: X \to Y'$ be a continuous embedding. Then the mapping $j: Y \to X'$, given by $\langle j(y), x\rangle = \langle y, i(x)\rangle$, is continuous. If, moreover, $i(X)$ is weak${}^*$-dense in $Y'$, then $j$ is injective, i.e. $j$ is a continuous embedding.

The proof is elementary.

Choose $X = H^m(\Omega)$ and $Y = L^1(\Omega)$ to obtain the situation you are asking for.

$\endgroup$
5
  • 1
    $\begingroup$ it might be instructive to just prove it directly using the imbedding of the Sobolev space into $L^\ifty$. $\endgroup$
    – Math604
    Dec 5, 2017 at 3:00
  • $\begingroup$ @ Jochen Glueck. Thank you so much for your answer. $\endgroup$
    – Jane
    Dec 5, 2017 at 9:08
  • $\begingroup$ @ Math604. Thank you so much. Could you please show a proof by using the imbedding of Sobolev space into $L^\infty$? $\endgroup$
    – Jane
    Dec 5, 2017 at 9:11
  • $\begingroup$ fix $f \in L^1(\Omega)$ and consider the linear functional $ T_f( \phi):= \int_\Omega f(x) \phi(x) dx$. Now try and show $ T_f \in (H^m)^*$; so do Holder on the integral with $ 1 $ and $ \infty$ and use imbedding $\endgroup$
    – Math604
    Dec 5, 2017 at 16:36
  • $\begingroup$ In fact, I have ever tried to prove in the same way. But I met a problem on one step. More precisely, for $f\in L^1$, define $T_f(\phi):=\int_\Omega f(x)\phi(x)dx$ $\forall \phi\in H^m$. It follows from $H^m\hookrightarrow L^\infty$ that $T_f\in (H^m)^*$ and $\|T_f\|\leq \|f\|_{L^1}$. Moreover, the mappling $T:f\longrightarrow T_f$ is a bijection from $L^1$ onto $T(L^1)$ which is a subset of $(H^m)^*$. We are left to prove that $\|T_f\|=\|f\|_{L^1}$ such that $T$ is a Isometric isomorphism. My question is how to prove that $\|f\|_{L^1}\leq \|T_f\|$? Thank you very much. $\endgroup$
    – Jane
    Dec 5, 2017 at 21:23

Your Answer

By clicking “Post Your Answer”, you agree to our terms of service and acknowledge you have read our privacy policy.

Not the answer you're looking for? Browse other questions tagged or ask your own question.